قدرت پمپ آب

مدیران انجمن: parse, javad123javad

ارسال پست
نمایه کاربر
rohamavation

نام: roham hesami radرهام حسامی راد

محل اقامت: 100 مایلی شمال لندن جاده آیلستون، لستر، لسترشر. LE2

عضویت : سه‌شنبه ۱۳۹۹/۸/۲۰ - ۰۸:۳۴


پست: 3288

سپاس: 5494

جنسیت:

تماس:

قدرت پمپ آب

پست توسط rohamavation »

پمپ آب را در نظر بگیرید که آب را از طریق یک لوله هل می دهد:
تصویر
ما می خواهیم انرژی مورد نیاز پمپ آب را برای حفظ جریان آب در لوله پیدا کنیم. نوشتن قانون دوم نیوتن برای آبی که از سطح مقطع استوانه ای آن خارج می شود،
$F= v \frac{dm}{dt}$
اکنون،$P = F \cdot v$
$P= v \frac{dm}{dt} v$
$\frac{dm}{dt} = \rho A v$
$P= v^3 \rho A$
حالا جواب واقعی ضریب نصف است، کجای این جا اشتباه کرده ام؟
من باید در مورد نحوه حرکت آب بیشتر بگم
سناریوی 1: جریان حالت پایدار
فرض کنید یک لوله طولانی با آب در جریان با سرعت ثابت v داشتید
. چقدر انرژی لازم است تا آب در آن سرعت جریان داشته باشد؟ البته، در غیاب تلفات ویسکوز یا آشفته (که در تمام این پاسخ فراموش کردم)، پاسخ فقط صفر است، درست مانند اینکه انرژی صفر برای نگه داشتن یک بلوک با سرعت ثابت روی یک سطح بدون اصطکاک لازم است.
بنابراین برای به دست آوردن یک پاسخ غیر ضروری، باید مشخص کنید که چگونه آب جدید از حالت استراحت به سرعت v آورده می شود
. یعنی باید به نحوه ورود آب به لوله فکر کنید.
سناریوی 2: جریان محافظه کارانه
فرض کنید انتهای لوله را در ظرفی پر از آب می چسبانیم. ما یا پمپی را راه اندازی می کنیم تا آب با سرعت مورد نظر از لوله عبور کند، یا اگر ظرف آب به اندازه کافی بالا باشد، می توانیم فقط از فشار هیدرواستاتیک موجود استفاده کنیم. ما باید انرژی صرف کنیم تا آب از طریق لوله جریان یابد، چه به عنوان انرژی الکتریکی در پمپ یا انرژی پتانسیل گرانشی در آب در ظرف. برای بتن بودن، فرض کنید لوله نزدیک بالای ظرف وصل شده است، بنابراین انرژی فقط از یک پمپ می آید.
اگر تلفات ویسکوز یا متلاطم وجود نداشته باشد، توانی که باید وارد کنید برابر با نرخ تغییر انرژی جنبشی آب است.
$P = \frac{dK}{dt} = \frac12 \frac{dm}{dt} v^2 = \frac12 \rho A v^3.$این نتیجه ای است که توسط چندین پاسخ دیگر ارائه شده است.
چرا استدلالم کار نمی کند؟
تا حدودی ظریف است. اول، همانطور که پاسخ های دیگر گفته اند، اگر یک پمپ را در ورودی لوله قرار دهید، در هنگام عبور آب سرعت آن را افزایش می دهد. بنابراین، شما نمی توانید از معادله استفاده کنید
$P_{\mathrm{pump}} = F_{\mathrm{pump}} v$زیرا سرعت آب در واقع به طور مداوم از صفر به v در حال افزایش است
. با این حال، این یک پاسخ کامل نیست، زیرا شما همچنین می توانید پمپ آب را در وسط لوله قرار دهید، جایی که آب از قبل با سرعت یکنواخت v در جریان است.
. در آن صورت، با فرض یک پمپ ایده آل، معادله شما $P_{\mathrm{pump}} = F_{\mathrm{pump}} v$ است
کاملا درست است
در اینجا مشکل در واقع اولین معادله شماست، که به طور ضمنی فرض می کند که نیروی پمپ تنها نیروی خارجی در سیستم است. به ورودی لوله فکر کنید. آب خارج از آن تقریبا با سرعت صفر حرکت می کند، در حالی که آب داخل با سرعت قابل توجهی حرکت می کند. بنابراین، بر اساس اصل برنولی، فشار در خارج از پمپ بیشتر است و این اختلاف فشار نیروی اضافی ایجاد می کند که به سرعت بخشیدن به آب کمک می کند. در واقع، برای هندسه‌های ایده‌آلی خاص، در واقع دقیقاً برابر با $F_{\mathrm{pump}}$ است
، که به این معنی است که $F_{\mathrm{pump}}$ فقط نیمی از نیروی خالص است. درج این ضریب 2
این ضریب 2 یک اثر شناخته شده در لوله کشی (یعنی دینامیک سیال اعمال شده) است که منجر به پدیده ورید منقبض می شود. به طور کلی در فیزیک مقدماتی ذکرنشده زیرا تا حدودی ظریف است، اما مهندسان دینامیک سیالات همیشه آن را به حساب می آورند، زیرا در غیر این صورت همه نتایج آنها کاملاً اشتباه خواهد بود.
سناریو 3: فرآیند ذاتا غیر کشسان
همچنین شرایطی وجود دارد که چیزی مانند پاسخ شما صحیح است. به عنوان مثال، فرض کنید یک فرورفتگی دایره ای بسیار طولانی دارید که آب با سرعت v از آن عبور می کند
. حال فرض کنید که باران روی ناودان می‌بارد و جرم آب را با سرعت dm/dt افزایش می‌دهد.تصویر
، در حالی که یک پمپ داخل آب آن را با سرعت ثابت نگه می دارد. در این صورت استنتاج شما صحیح خواهد بود، زیرا هیچ نیروی خارجی مهمی به جز نیروی پمپ وجود ندارد، بنابراین
$P = \frac{dm}{dt} \, v^2$بدون ضریب 1/2 . از سوی دیگر، با صرفه جویی در انرژی داریم $P = \frac12 \frac{dm}{dt} \, v^2 + \frac{dE_{\mathrm{int}}}{dt}$جایی که $E_{\mathrm{int}}$
انرژی درونی آب است. این دو عبارت سازگار هستند اگر
$\frac{d E_{\mathrm{int}}}{dt} = \frac12 \frac{dm}{dt} \, v^2.$اما آیا من قبلاً نگفتم که همه تلفات چسبناک و متلاطم را نادیده می گیریم؟ خوب، در این مورد، شما نمی توانید. اگر ویسکوزیته یا تلاطم وجود نداشت، آب باران فقط در بالای f قرار می گرفت.
آب را کم می کند و پمپ به هیچ وجه به برق نیاز ندارد. نوعی اتلاف برای افزایش سرعت باران مورد نیاز است
. از آنجایی که این برخورد اساساً یک برخورد غیرکشسان است، انرژی را هدر می دهد و قدرت پمپ را دو برابر بیشتر از نتیجه ساده می کند. این یکی دیگر از فاکتورهای معروف 2 است
(مرتبط با قبلی نیست) که به طور منظم در مسائل پیچیده فیزیک دبیرستان نشان داده می شود.تصویر
تصویر
جواب عالی من با توضیحات مشکل سردرگمی زیادی داشتم. برای جلوگیری از مشکلات استفاده از یک پمپ واقعی، من توصیف شما را در مورد لوله متصل به پایین یک مخزن بزرگ مایع دوست دارم. در اینجا "پمپ" سر مایع است. نیروی اعمال شده بر عنصر بی نهایت کوچک جرم dm توسط پمپ خواهد بود $\mathrm dF=\mathrm d m \: a =\mathrm dm \frac{\mathrm dv}{\mathrm dt}$
اکنون می توانید dm را تغییر دهید و dv بدست آوردن $\mathrm dF=\mathrm dv \frac{\mathrm dm}{\mathrm dt}$
اینجا $dm/dt$ و$\rho A v_0$ است
جایی که $v_0$ سرعت نهایی خروج آب است و بنابراین ثابت است. بنابراین بیان نیرو خواهد بود
$\mathrm dP=\mathrm dF \: v$ اکنون، قدرت اعمال شده به عنصر بینهایت کوچک در هر لحظه خواهد بود
$\mathrm dP=\mathrm dF \: v$ جایی که v سرعت عنصر در آن لحظه است. بنابراین پس از ادغام، دریافت می کنید
$\int \mathrm dP=\rho A v_0 \int_0^{v_0}v\:\mathrm dv$ بنابراین شما دریافت می کنید$\boxed{P=\frac 1 2 \rho A v_0^3}$ این سوال به دنبال ارزیابی قدرت پمپ برای یک مورد بسیار خاص است: جریان ثابت در یک لوله افقی بدون تلفات اصطکاک که در آن سرعت ورودی کم است، و جایی که تمام کار پمپ صرف نظر از نرخ جریان به ایجاد اختلاف فشار ثابت می‌شود. قدرت پمپ برای این مورد خاص، قدرت پمپ برای یک شبکه جریان به طور کلی نیست، به ویژه با توجه به تغییر قدرت پمپ با رفتار نرخ جریان برای پمپ گریز از مرکز رایج. بنابراین، من یک بحث کلی در مورد "جریان کاری" و کاربرد معادله برنولی ارائه می کنم. سپس، در پایان این پاسخ، از نتایج کلی برای پرداختن به این مورد خاص استفاده می‌کنم و به همان نتیجه‌ای می‌رسم که سایر پاسخ‌دهندگان به این سؤال هستند. به طور خاص، برای توضیح 1/2 در قدرت پمپ من بحث کلی نسبتاً طولانی را برای تأکید بر اینکه نتیجه قدرت پمپ در مورد خاص به طور کلی معتبر نیست، اضافه می کنم.
در موقعیتی از لوله که سطح مقطع A است و سرعت سیال V است ، نرخ جریان انرژی در واحد جرم$e_f \dot m_f$ است
جایی که $e_f$ (J/kg) انرژی در واحد جرم جرم جاری$m_f$ است (کیلوگرم)، و$\dot m_f$ (kg/sec) سرعتی است که در آن جرم از سطح مقطع A عبور می کند
. همانطور که در شکل زیر نشان داده شده است، $\rho AV$ برابر $\rho AV$ است جایی که$ ρ$ چگالی است؛ A و V می تواند توابع موقعیت باشد. $e_f = h + {V^2 \over 2} + zg$
(J/kg) که در آن h آنتالپی در واحد جرم، V سرعت، z ارتفاع، و g شتاب گرانش "جریان کاری" همراه با فشار دادن جرم در عرض یک مرز وجود دارد. آنتالپی در واحد جرم به صورت $h = u + p/\rho$ تعریف می شود $u$ انرژی داخلی در واحد جرم و p فشار است $p/\rho$
این اصطلاح گاهی اوقات "جریان کار" نامیده می شود زیرا کار در واحد جرم است که در مرز توسط سیال زیر $\Delta m_f$ انجام می شود.
برای فشار دادن$\Delta m_f$ از طریق فاصله $V\Delta t$
.برای جزئیات نگاه کنید به کتاب خوبی در مورد ترمودینامیک پایه، مانند ترمودینامیک نوشته اوبرت، که در آن قانون اول ترمودینامیک برای یک سیستم باز توسعه داده شده است، به عنوان سیستمی که جرم می تواند وارد سیستم شود و از آن خارج شود.تصویر
جریان منطقه
مهم است که بفهمیم بحث قبلی در مورد جریان جرم در یک مرز ثابت است. در بیشتر کاربردها، ما با تغییر انرژی یک سیستم (یک سیستم ترمودینامیکی باز) سروکار داریم که به عنوان ناحیه ای بین مرزها تعریف می شود که جرم می تواند از طریق آن جریان یابد. سیستمی با مرز 1 که جرم وارد می شود و مرز 2 که جرم خارج می شود را در نظر بگیرید. "جریان کار" در واحد جرم انجام شده توسط محیط اطراف در سیستم به دلیل فشار سیال در محیط اطراف به داخل سیستم در مرز 1 $p_1/\rho_1$ است.
. "جریان کار" در واحد جرم انجام شده توسط سیستم در محیط اطراف به دلیل فشار سیال در سیستم به محیط اطراف به عنوان مرز 2 $p_2/\rho_2$ است.
. انرژی در واحد جرم وارد شده به سیستم در مرز 1 $e_{f_\enspace 1}$است و انرژی در واحد جرم خروجی از سیستم در مرز 2 $e_{f_\enspace 2}$ است
.توان پمپ میزان کار انجام شده توسط پمپ بر روی یک سیستم است. قدرت پمپ هم به مشخصات سر پمپ و هم به سر سیستم برای عناصر جریان (لوله ها و اتصالات) که پمپ تامین می کند بستگی دارد. هد پمپ می تواند به دبی و دبی به هد سیستم بستگی داشته باشد. با فرض جریان حالت پایدار برای یک مایع تراکم ناپذیر، قانون اول ترمودینامیک برای یک سیستم جریان پایدار را می توان با استفاده از معادله برنولی بیان کرد. سر دارای واحدهای فاصله (متر) است و به سادگی هر عبارت در معادله برنولی بر حسب متر بیان می شود.
سیستم پمپاژ نشان داده شده در شکل زیر را با استفاده از یک پمپ گریز از مرکز در نظر بگیرید. پمپ در نقطه‌ای کار می‌کند که با محل تقاطع منحنی سر پمپ با منحنی سر سیستم تعیین می‌شود. برای یک پمپ گریز از مرکز، هد پمپ $h_{pump}$ با دبی حجمی Q کاهش می یابد (متر مکعب در ثانیه) و هد سیستم با Q افزایش می یابد
به دلیل تلفات اصطکاک در شبکه جریان. کار در واحد جرم $W_{mass}$
(J/kg) انجام شده توسط پمپ برابر $gh_{pump}$ است
$h_{pump} = h_{system}$ جایی که g
شتاب گرانش، $h_{pump}$ است
سر پمپ در نقطه عملیاتی و سیستم است
سر سیستم در نقطه عملیاتی است. نیروی پمپ$P_{pump} = \rho Qgh_{pump} = \rho Qgh_{system}$ست
(J/sec) که در آن $\rho$
چگالی مایع است. بنابراین برای ارزیابی توان پمپ نقطه عملیاتی مورد نیاز است و این به ویژگی های پمپ و سیستم جریانی که پمپ تامین می کند بستگی دارد. توان «ترمز» نیرویی است که توسط (مثلاً یک موتور یا یک توربین) به پمپ عرضه می شود و برابر است با توان پمپ تقسیم بر بازده پمپ. حال مورد خاص سیستمی را در نظر بگیرید که به عنوان جریان ثابت در یک لوله افقی بدون تلفات اصطکاک در جایی که سرعت ورودی کم است تعریف می شود. برای جلوگیری از مشکلات مربوط به عملکرد یک پمپ واقعی، "پمپ" در اینجا یک سر مایع در یک مخزن بزرگ با لوله در پایین مخزن است معادله برنولی برای سیستم ${p_1 \over \rho} + {V_1^2 \over 2} = {p_2 \over \rho} + {V_2^2 \over 2}$ است.تصویر [/img]
. برای سرعت ناچیز $V_1$
ما ${(p_1 - p_2) \over \rho} = {V_2^2 \over {2}}$داریم
(J/kg). نرخ جریان جرمی $\rho AV_2$ است
(kg/sec) بنابراین توان ${V_2^2 \over {2}} \rho AV_2 = {1 \over 2} \rho A V_2^3$ است
(J/sec یا وات) به دلیل اختلاف فشار $p_1 - p_2$ . این اختلاف فشار توسط سر مایع ایجاد می شود. همانطور که قبلاً گفتم، این توان برای این مورد خاص است و به طور کلی توان پمپ برای یک شبکه جریان نیست.توان را می توان با در نظر گرفتن "جریان کاری" نیز ارزیابی کرد. کار جریان خالص به ازای واحد جرم انجام شده روی سیال در لوله (جریان منهای جریان خارج)${(p_1 - p_2) \over \rho}$ است.
(J/kg)، و همانطور که قبلا نشان داده شد، برای مورد خاص مورد بررسی، این منجر به قدرت پمپ${1 \over 2} \rho A V_2^3$ می شود.
.قدرت میزانی است که برای افزودن انرژی به آن نیاز دارید. در هر بازه زمانی Δt ، شما باید یک لجن جدید آب Δm را شتاب دهید
از صفر تا سرعت v که به انرژی$\Delta E=\frac{1}{2}\Delta m v^2$ نیاز دارد . اکنون$\Delta m=\rho A v \Delta t$ ، بنابراین
$\frac{\Delta E}{\Delta t}=\frac{1}{2}\rho A v^3.$
در مورد اینکه در کجای استخراجم خود اشتباه کردم، استفاده نادرست از $P=F\cdot v$ است
. این عبارت در صورتی معتبر است که v در دوره ای که F ثابت است اعمال می شود. در اینجا اینطور نیست: F جرم تفاضلی آب dm را تسریع می کند
از 0 تا v ; سرعت متوسط آن در طول این فرآیند v/2 است
.برای بررسی کار خود، به یاد داشته باشید که توان حاصلضرب یک متغیر تلاش (فشار در این مورد) و یک متغیر جریان (سرعت جریان جرمی در این مورد) است، و مراقب باشید که واحدها ثابت باشند. اگر فشار منبعی را که بر روی جرم جاری اعمال می‌کند، بدانید، قدرت آن را می‌دانید.
بنابراین، هرگز قدرت نیروی ضربدر فاصله تقسیم بر زمان را دست کم نگیرید. شما باید این کار را به این ترتیب انجام دهید تا واحدها درست بیرون بیایند.
مشکل از کلاس های ابتدای در دبیرستان (بدون حساب ... وقتی به دانش آموز حساب دیفرانسیل و انتگرال و بردار آموزش داده نمی شود). نیرو برای فنراست
$F=-kx$پس کار فنرچیست$=kx^2$W=-Fx$$
فاکتور2 کجاست عامل؟ اما اگر نوشته بودم
$W=\int \vec F\cdot \mathrm d(-\vec x)$بیش از آن که بتوانید نتیجه مورد انتظار را دریافت کنید.
بیایید با قانون دوم نیوتن شروع کنیم.
$\vec F=\frac{d\vec p}{dt}$
$=m\vec a+\vec v \frac{dm}{dt}$
$m=\rho Ax_0$
$\frac{dm}{dt}=\rho Av_0$
$P=\int\frac{dW}{dt}=\int \vec F\cdot d\vec v$
$=\int_0^{v_0} [m\frac{d\vec v}{dt}+\vec v \frac{dm}{dt}] \cdot \mathrm d\vec v$
بنابراین dm/dt مستقل از v است
بنابراین می توانید آن را ثابت در نظر بگیرید. حتی ممکن است بپرسید چرا یک ترم اضافی وجود دارد؟ اگر آب با سرعت ثابت جریان نداشته باشد، اضافی معتبر است.
حفظ مسئله تکانه با پمپ فرض کنید سناریوی زیر را داریم:
تصویر
یک مورد بسیار ساده که در آن ما یک لوله داریم که به یک مخزن بزرگ (پر از آب) می رود و یک پمپ آب را به بیرون (از چپ به راست) فشار می دهد.
اولین چیزی که به ذهن می رسد استفاده از معادله برنولی است که در صورت وجود پمپ به صورت:$P_1 + P_p + \frac{1}{2} \rho v_{1}^2 + \rho g h_1 = P_2 + \frac{1}{2} \rho v_{2}^2 + \rho g h_2$بیانن میکنم در اینجا $P_p$ اختلاف فشار ناشی از پمپ است. می‌توانیم معادله قبلی را به صورت $P_p = \frac{1}{2} \rho v_{2}^2$ ساده کنیم
از آنجایی که اختلاف ارتفاع وجود ندارد، اختلاف فشار وجود ندارد $P_1 = P_2 = P_{ATM}$ و سرعت اولیه در مخزن 0 است
.تا اینجا که سعی کردم نیروی اعمال شده توسط پمپ را پیدا کنم، خوب است، در این مورد می گویم$F_p = P_p A_p = \frac{1}{2} \rho A_p v_{2}^2$
و با اعمال پایستگی تکانه نیروی وارد شده توسط آب $|F_w| = \dot{m}v_2 = \rho A_2 v_{2}^2 = |F_p|$ که به این معنی است که $A_p = 2 A_2$
.این قسمتی است که معنی ندارد، به نظر می رسد که ناحیه ای که فشار پمپ در آن اعمال می شود باید دو برابر فشار ناحیه خروجی باشد، گویی "مجبور" شده ام پمپ را در یک منطقه خاص قرار دهم. شاید اتفاقی که می افتد این است که پمپ در جای دیگری (جایی که منطقه$2A_2$ است) نیرو وارد می کند.
که در جریان لغزشی در مخزن قرار می گیرد، به نوعی مانند یک فن محوری) اما پس از آن یک گرادیان فشار در سراسر محل واقعی پمپ وجود نخواهد داشت. واضح است که من چیزی را از دست داده ام.
تعیین شما از افزایش فشار در سراسر پمپ صحیح است:
$P_p=\Delta P=\frac{1}{2}\rho v_2^2$تعادل حرکت افقی روی پمپ به تنهایی به شرح زیر است:
$-\Delta P_pA_2+F_p=\rho v_2A_2(v_2-v_2)=0$یا
$F_p=\frac{1}{2}\rho v_2^2A_2$با فرض اینکه نواحی لوله ورودی و خروجی به پمپ هر دو A2 هستند
. این نیروی افقی که پمپ بر روی آب جاری وارد می کند شامل نیروی آب روی محفظه پمپ به اضافه نیروی آب وارد بر پره های پمپ می شود. این برخلاف نیروی افقی خالصی است که آب بر پمپ وارد می کند (که در جهت x منفی است).
به نظر من مدل من سازگار نیست. با نادیده گرفتن تمام اثرات ویسکوزیته و فرض آب به عنوان یک سیال تراکم ناپذیر با چگالی ثابت، در لوله افقی مستقیم با مساحت ثابت، تنها اثر پمپ افزایش فشار در آن است، در حالی که سرعت برای تعادل جرم یکنواخت است. اگر اینطور است، $P_2>P_1= P_{atm}$
و من حدس می‌زنم که خطوط جریان آب جت مستقیم نیستند، بلکه به دلیل اختلاف فشار $P_2 >P_{atm}$، یک شکل مخروطی کیفی در انتهای 2 لوله ایجاد می‌کنند.
.این یک توصیف بسیار کیفی از مشکل است، در حالی که برای توصیف دقیق‌تر میدان جریان و میدان فشار، حدس می‌زنم که به مدل دقیق‌تری (احتمالاً با حل عددی) یا اندازه‌گیری نیاز دارید.
تصویر

ارسال پست